Thứ năm, 02/05/2024
IMG-LOGO

Câu hỏi:

05/07/2022 106

Tập nghiệm của bất phương trình\[{\log _2}\left( {x\sqrt {{x^2} + 2} + 4 - {x^2}} \right) + 2x + \sqrt {{x^2} + 2} \le 1\] là \(\left( { - \sqrt a ; - \sqrt b } \right)\).Khi đó abab bằng

A.\[\frac{{12}}{5}\]

B. \[\frac{5}{{12}}\]

C. \[\frac{{15}}{{16}}\]

D. \[\frac{{16}}{{15}}\]

Đáp án chính xác

Trả lời:

verified Giải bởi qa.haylamdo.com

Điều kiện :

\[x\sqrt {{x^2} + 2} + 4 - {x^2} > 0 \Leftrightarrow x\left( {\sqrt {{x^2} + 2} - x} \right) + 4 > 0 \Leftrightarrow x.\frac{2}{{\sqrt {{x^2} + 2} + x}} + 4 > 0\]

\[ \Leftrightarrow \frac{{2x}}{{\sqrt {{x^2} + 2} + x}} + \frac{{4\left( {\sqrt {{x^2} + 2} + x} \right)}}{{\sqrt {{x^2} + 2} + x}} > 0 \Rightarrow 6x + 4\sqrt {{x^2} + 2} > 0\] (vì \[\sqrt {{x^2} + 2} > x;\,\forall x\])

\[ \Leftrightarrow 2\sqrt {{x^2} + 2} > - 3x \Leftrightarrow \left[ {\begin{array}{*{20}{c}}{ - 3x < 0}\\{\left\{ {\begin{array}{*{20}{c}}{ - 3x \ge 0}\\{4({x^2} + 2) > {{( - 3x)}^2}}\end{array}} \right.}\end{array}} \right.\]</>

\( \Leftrightarrow \left[ {\begin{array}{*{20}{c}}{x > 0}\\{\left\{ {\begin{array}{*{20}{c}}{x \le 0}\\{5{x^2} < 8}\end{array}} \right.}\end{array}} \right. \Leftrightarrow \left[ {\begin{array}{*{20}{c}}{x > 0}\\{ - \frac{{\sqrt {40} }}{5} < x \le 0}\end{array}} \right.\)

Khi đó ta có\[{\log _2}\left( {x\sqrt {{x^2} + 2} + 4 - {x^2}} \right) + 2x + \sqrt {{x^2} + 2} \le 1\]

\[\begin{array}{*{20}{l}}{ \Leftrightarrow {{\log }_2}\left( {x\left( {\sqrt {{x^2} + 2} - x} \right) + 4} \right) + 2x + \sqrt {{x^2} + 2} \le 1}\\{ \Leftrightarrow {{\log }_2}\left( {\frac{{2x}}{{\sqrt {{x^2} + 2} + x}} + 4} \right) + 2x + \sqrt {{x^2} + 2} \le 1}\\{ \Leftrightarrow {{\log }_2}\left( {\frac{{6x + 4\sqrt {{x^2} + 2} }}{{\sqrt {{x^2} + 2} + x}}} \right) + 2x + \sqrt {{x^2} + 2} \le 1}\end{array}\]

\[\begin{array}{l} \Leftrightarrow lo{g_2}(6x + 4\sqrt {{x^2} + 2} ) - lo{g_2}(\sqrt {{x^2} + 2 + x} ) + 2x + \sqrt {{x^2} + 2} \le 1\\ \Leftrightarrow lo{g_2}[2(3x + 2\sqrt {{x^2} + 2} )] - lo{g_2}(\sqrt {{x^2} + 2} + x) + 2x + \sqrt {{x^2} + 2} \le 1\\ \Leftrightarrow lo{g_2}2 + lo{g_2}(3x + 2\sqrt {{x^2} + 2} ) - lo{g_2}(\sqrt {{x^2} + 2} + x) + 2x + \sqrt {{x^2} + 2} \le 1\\ \Leftrightarrow 1 + lo{g_2}(3x + 2\sqrt {{x^2} + 2} ) - lo{g_2}(\sqrt {{x^2} + 2} + x) + 2x + \sqrt {{x^2} + 2} \le 1\\ \Leftrightarrow lo{g_2}(3x + 2\sqrt {{x^2} + 2} ) + 3x + 2\sqrt {{x^2} + 2} \le lo{g_2}(\sqrt {{x^2} + 2} + x) + x + \sqrt {{x^2} + 2} ( * )\end{array}\]

Xét hàm số \[f\left( t \right) = t + {\log _2}t\,\] với t>0 ta có \[f'\left( t \right) = 1 + \frac{1}{{t.\ln 2}} > 0;\,\forall t > 0\]  nên f(t) là hàm đồng biến trên\[\left( {0; + \infty } \right)\]Từ đó

\[\begin{array}{l}( * ) \Leftrightarrow f(3x + 2\sqrt {{x^2} + 2} ) \le f(\sqrt {{x^2} + 2} + x)\\ \Leftrightarrow 3x + 2\sqrt {{x^2} + 2} \le \sqrt {{x^2} + 2} + x\\ \Leftrightarrow \sqrt {{x^2} + 2} \le - 2x\end{array}\]

\( \Leftrightarrow \left\{ {\begin{array}{*{20}{c}}{ - 2x \ge 0}\\{{x^2} + 2 \le 4{x^2}}\end{array}} \right. \Leftrightarrow \left\{ {\begin{array}{*{20}{c}}{x \le 0}\\{3{x^2} \ge 2}\end{array}} \right. \Leftrightarrow \left\{ {\begin{array}{*{20}{c}}{x \le 0}\\{\left[ {\begin{array}{*{20}{c}}{x \ge \frac{{\sqrt 6 }}{3}}\\{x \le - \frac{{\sqrt 6 }}{3}}\end{array}} \right.}\end{array}} \right. \Leftrightarrow x \le - \frac{{\sqrt 6 }}{3}\)

Kết hợp điều kiện \(\left[ {\begin{array}{*{20}{c}}{x > 0}\\{ - \frac{{\sqrt {40} }}{5} < x \le 0}\end{array}} \right.\) ta có\[ - \frac{{\sqrt {40} }}{5} < x \le - \frac{{\sqrt 6 }}{3}\] hay\[ - \sqrt {\frac{8}{5}} < x \le - \sqrt {\frac{2}{3}} \]

Tập nghiệm bất phương trình\[S = \left( { - \sqrt {\frac{8}{5}} ; - \sqrt {\frac{2}{3}} } \right]\] nên\[a = \frac{8}{5};b = \frac{2}{3} \Rightarrow a.b = \frac{8}{5}.\frac{2}{3} = \frac{{16}}{{15}}.\]

Đáp án cần chọn là: D

Câu trả lời này có hữu ích không?

0

CÂU HỎI HOT CÙNG CHỦ ĐỀ

Câu 1:

Có tất cả bao nhiêu giá trị nguyên của y sao cho tương ứng với mọi y luôn tồn tại không quá 63 số nguyên x thỏa mãn điều kiện \[{\log _{2020}}\left( {x + {y^2}} \right) + {\log _{2021}}\left( {{y^2} + y + 64} \right) \ge {\log _4}\left( {x - y} \right)\]

Xem đáp án » 05/07/2022 188

Câu 2:

Tập nghiệm của bất phương trình \[\log \left( {{x^2} + 25} \right) > \log \left( {10x} \right)\] là:

Xem đáp án » 05/07/2022 139

Câu 3:

Cho phương trình \[{11^x} + m = {\log _{11}}\left( {x - m} \right)\] với m là tham số. Có bao nhiêu giá trị nguyên của \[m \in \left( { - 205;205} \right)\;\] để phương trình đã cho có nghiệm?

Xem đáp án » 05/07/2022 134

Câu 4:

Xét bất phương trình \[\log _2^22x - 2\left( {m + 1} \right){\log _2}x - 2 < 0\]. Tìm tất cả các giá trị của tham số m để bất phương trình có nghiệm thuộc khoảng \[\left( {\sqrt 2 ; + \infty } \right).\]

Xem đáp án » 05/07/2022 123

Câu 5:

Tập nghiệm của bất phương trình \[{9^{\log _9^2x}} + {x^{{{\log }_9}x}} \le 18\]là:

Xem đáp án » 05/07/2022 112

Câu 6:

Bất phương trình  \[{\log _{\frac{4}{{25}}}}(x + 1) \ge {\log _{\frac{2}{5}}}x\] tương đương với bất phương trình nào dưới đây?

Xem đáp án » 05/07/2022 109

Câu 7:

Giải bất phương trình \[{\log _2}\left( {3x - 1} \right) \ge 3\]

Xem đáp án » 05/07/2022 108

Câu 8:

Tập hợp nghiệm của bất phương trình \(\)\[{\log _{\frac{1}{3}}}\left( {{x^2} - 2x + 1} \right) < {\log _{\frac{1}{3}}}\left( {x - 1} \right)\] là:

Xem đáp án » 05/07/2022 107

Câu 9:

Số nguyên nhỏ nhất thỏa mãn \[{\log _2}\left( {5x - 3} \right) > 5\] là:

Xem đáp án » 05/07/2022 104

Câu 10:

Xác định tập nghiệm S của bất phương trình \[\ln {x^2} > \ln \left( {4x - 4} \right)\]

Xem đáp án » 05/07/2022 104

Câu 11:

Cho hàm số y=f(x). Hàm số y=f′(x) có đồ thị như hình bên. Biết \[f\left( { - 1} \right) = 1,f( - \frac{1}{e}) = 2.\]. Tìm tất cả các giá trị của m để bất phương trình \[f(x) < ln( - x) + m\;\] nghiệm đúng với mọi \[x \in ( - 1; - \frac{1}{e}).\]

Cho hàm số y=f(x). Hàm số y=f′(x) có đồ thị như hình bên. Biết  (ảnh 1)

Xem đáp án » 05/07/2022 104

Câu 12:

Cho \[m = {\log _a}\sqrt {ab} \] với a,b>1 và \[P = \log _a^2b + 54{\log _b}a\]. Khi đó giá trị của m để P đạt giá trị nhỏ nhất là:

Xem đáp án » 05/07/2022 101

Câu 13:

Tập nghiệm của bất phương trình \[{\left( {\sqrt 5 - 2} \right)^{\frac{{2x}}{{x - 1}}}} \le {\left( {\sqrt 5 + 2} \right)^x}\] là:

Xem đáp án » 05/07/2022 99

Câu 14:

Tập nghiệm của phương trình \[{\log _3}\left( {{{\log }_{\frac{1}{2}}}x} \right) < 1\] là

Xem đáp án » 05/07/2022 98

Câu 15:

Tìm tập nghiệm S của bất phương trình \[{\log _{\frac{1}{2}}}\left( {x - 1} \right) > {\log _{\frac{1}{2}}}\left( {5 - 2x} \right)\]

Xem đáp án » 05/07/2022 95

Câu hỏi mới nhất

Xem thêm »
Xem thêm »